Find the missing angle. Round your answer to the nearest tenth. PLS HURRY.

Answers

Answer 1

Answer:

x = 24.4°

Hope this helps... Have a good day!!


Related Questions

Grandma is making a quilt. She has 540 cm of fabric to border the quilt. What is the greatest possible area for the quilt?
Question 1 options:


11 664 cm^2


18225 cm^2


72900 cm^2


291600 cm^2

Show your work:

Answers

Answer:

18225 cm²

Step-by-step explanation:

Divide 540 by 4 to get the length of all sides

540/4 = 135

Square 135 to get the max possible size

135² = 18225

18225 cm²  is the greatest possible area for the quilt.

What is area?

The measurement that expresses the size of a region on a plane or curved surface is called area. Surface area refers to the area of an open surface or the boundary of a three-dimensional object, whereas the area of a plane region or plane area refers to the area of a form or planar lamina.

Given

Divide 540 by 4 to obtain the length of all sides

540/4 = 135

Square 135 to acquire the max possible size

135² = 18225

18225 cm²  is the greatest possible area for the quilt.        

To learn more about area refer to:

https://brainly.com/question/25292087

#SPJ2

Three red balls, 5 green balls and a number of blue balls are put together in a sac. One ball is picked at random from the sac. If the probability of picking a red ball is 1|6, find the a) The number of blue balls in sac. B) the probability of picking a green ball​

Answers

Answer:

total balls = 18 .... 3/x = 1/6

blue = 10 ... 18-(5+3) = 10

p of green = 5/18 = .277

Step-by-step explanation:

Convert to decimal degrees.

-(167° 31”)

[?]°

Enter your answer with three decimal places.

Answers

Answer:

The angle in decimal form is 167.009°.

Step-by-step explanation:

We know an angle in terms of integer angles, minutes and seconds, whose conversion into decimal degrees is expressed by the following formula:

[tex]\theta = n + \frac{m}{60}+\frac{s}{3600}[/tex] (1)

Donde:

[tex]n[/tex] - Integer angle, in sexagesimal degrees.

[tex]m[/tex] - Minutes.

[tex]s[/tex] - Seconds.

If we know that [tex]n = 167[/tex], [tex]m = 0[/tex] and [tex]s = 31''[/tex], then the angle in decimal form is:

[tex]\theta = 167^{\circ}+\frac{0}{60}^{\circ} + \frac{31}{3600}^{\circ}[/tex]

[tex]\theta = 167.009^{\circ}[/tex]

The angle in decimal form is 167.009°.

Find the length of the segment indicated. Round your answer to the nearest 10th if necessary.

Answers

Answer:

x=13.6

Step-by-step explanation:

By Pythagoras theorem, 5.5^2+x^2=14.7^2. x^2=14.7^2-5.5^2. x=13.6

Find cosθ+cos3θ+cos5θ+cos7θ by using the Sum-to-Product Formula.
Please also show your work as well. Thanks!

Answers

Answer:

[tex] \rm\displaystyle 4\cos( \theta) \cos \left( {2\theta} \right) \cos \left( {4 \theta } \right) [/tex]

Step-by-step explanation:

I assume the question want us to rewrite cosθ+cos3θ+cos5θ+cos7θ by using Sum-to-Product Formula and note that it's not an equation therefore θ can never be specified

===========================

so we want to rewrite cosθ+cos3θ+cos5θ+cos7θ by using Sum-to-Product Formula the good news is that the number of the function of the given expression is even so there's a way to do so, rewrite the expression in parentheses notation:

[tex] \rm\displaystyle \left( \cos( \theta) + \cos(3 \theta) \right) + \left(\cos(5 \theta) + \cos(7 \theta) \right)[/tex]

recall that,Sum-to-Product Formula of cos function:

[tex] \rm \boxed{\displaystyle \cos( \alpha ) + \cos( \beta ) = 2 \cos \left( \frac{ \alpha + \beta }{2} \right) \cos \left( \frac{ \alpha - \beta }{2} \right) }[/tex]

notice that we have two pair of function with which we can apply the formula thus do so,

[tex] \rm\displaystyle \left( 2\cos \left( \frac{ \theta + 3 \theta}{2} \right)\cos \left( \frac{ \theta - 3 \theta}{2} \right) \right) + \left(2\cos \left( \frac{5 \theta + 7 \theta}{2} \right) \cos \left( \frac{5 \theta - 7 \theta}{2} \right) \right)[/tex]

simplify addition:

[tex] \rm\displaystyle \left( 2\cos \left( \frac{4 \theta}{2} \right)\cos \left( \frac{ - 2\theta }{2} \right) \right) + \left(2\cos \left( \frac{12 \theta }{2} \right) \cos \left( \frac{ - 2 \theta}{2} \right) \right)[/tex]

simplify division:

[tex] \rm\displaystyle \left( 2\cos \left( {2 \theta} \right)\cos \left( { - \theta } \right) \right) + \left(2\cos \left( {6 \theta } \right) \cos \left( { - \theta} \right) \right)[/tex]

By Opposite Angle Identities we acquire:

[tex] \rm\displaystyle \left( 2\cos \left( {2 \theta} \right)\cos \left( { \theta } \right) \right) + \left(2\cos \left( {6 \theta } \right) \cos \left( { \theta} \right) \right)[/tex]

factor out 2cosθ:

[tex] \rm\displaystyle 2 \cos( \theta) (\cos \left( {2 \theta} \right) + \cos \left( {6 \theta } \right) )[/tex]

once again apply Sum-to-Product Formula which yields:

[tex] \rm\displaystyle 2 \cos( \theta) (2\cos \left( {4\theta} \right) \cos \left( {2 \theta } \right) )[/tex]

distribute:

[tex] \rm\displaystyle 4\cos( \theta) \cos \left( {2\theta} \right) \cos \left( {4 \theta } \right) [/tex]

and we're done!

Find cos 0
A. 15/8
B. 15/17
C. 8/15
D. 8/17

Answers

Answer:

A.15/8

Step-by-step explanation:

the answer is 15/8

Answer:

D.

[tex]{ \tt{ \cos( \theta) = \frac{adjacent}{hypotenuse} }} \\ \\ { \tt{ \cos( \theta) = \frac{8}{ \sqrt{ {15}^{2} + {8}^{2} } } }} \\ \\ { \tt{ \cos( \theta) = \frac{8}{ \sqrt{289} } }} \\ \\ { \tt{ \cos( \theta) = \frac{8}{17} }}[/tex]

If f is continuous for all x, which of the following integrals necessarily have the same value?

Answers

Answer:

B

Step-by-step explanation:

Given the three integrals, we want to determine which integrals necessarily have the same value.

We can let the first integral be itself.

For the second integral, we can perform a u-substitution. Let u = x + a. Then:

[tex]\displaystyle du = dx[/tex]

Changing our limits of integration:

[tex]u_1=(0)+a=a \text{ and } u_2 = (b+a)+a = b+2a[/tex]

Thus, the second integral becomes:

[tex]\displaystyle \int_{0}^{b+a}f(x+a)\, dx = \int_a^{b+2a} f(u)\, du[/tex]

For the third integral, we can also perform a u-substitution. Let u = x + c. Then:

[tex]\displaystyle du = dx[/tex]

And changing our limits of integration:

[tex]\displaystyle u_1=(a-c)+c=a \text{ and } u_2=(b-c)+c=b[/tex]

Thus, our third integral becomes:

[tex]\displaystyle \int_{a-c}^{b-c}f(x+c)\, dx = \int_{a}^{b} f(u)\, du[/tex]

Since the only difference between f(x) and f(u) is the variable and both the first and third integral have the same limits of integration, our answer is B.

A square has a side length of 36 feet. This square is dilated by a scale factor of 2/3 to create a new square. What is the side length of the new square?

Answers

Answer:

24

Step-by-step explanation:

Multiply the side length by the dilation

36 x 2/3

72/3

Simplify

72/3 = 24

Your answer is correct

If the speed of an object in motion is doubled, its kinetic energy becomes how many times the original kinetic energy

Answers

Answer: Becomes four times

Step-by-step explanation:

Given

Speed is doubled for a moving object

Suppose initial speed is u

Increased speed is 2u

Kinetic Energy is given by

[tex]\Rightarrow K=0.5mu^2[/tex]

When speed is doubled

[tex]\Rightarrow K'=0.5m(2u)^2\\\Rightarrow K'=(0.5mu^2)\times 4\\\Rightarrow K'=4K[/tex]

Kinetic energy becomes four times

cos theta / sec theta -1 - sin theta / 1+cos theta = 2 cot theta​

Answers

Step-by-step explanation:

Explanation is in the attachment

Hope it is helpful to you

An angle measures 73.6° less than the measure of its supplementary angle. What is the measure of each angle?

Answers

Answer:

Smaller angle = 53.2

Larger angle = 126.8

Step-by-step explanation:

Lets say x is the measure of the supplement. Since we know they're supplementary, we know their angle measure sum will equal 180. We can set up our equation like this [tex]x + (x-73.6) = 180[/tex]. Note: (x - 73.6) is the measure of the smaller angle. By solving, we get 126.8 degrees for the measure of the supplement. If we plug in the value of x into (x-73.6), we get 53.2 degrees as the angle measure of the smaller angle.

Please help! Identify an equation in point-slope form for the line parallel to y=3/4x-4 that passes through (-1,7).

Answers

The answer is c y-7=3/4(x+1) hope this helps.

State what additional information is required in order to know that the triangle in the image below are congruent for the reason given…

Reason: HL Postulate

Answers

Answer:

FG ≈ FL (Both are hypotenuse, supposed to be equal in order to the congruency to become HL)

Answered by GAUTHMATH

A house on the market was valued at $472,000. After several years, the value increased by 19%. By how much did the house's value increase in dollars? What is the current value of the house?

Answers

Step-by-step explanation:

Increase in dollars

19/100 x 472.000 = $89,670

and the current value house is $472,000 + $89,670 = $561,680

The difference between 15 and 9 is subtracted from 5 times the sum of 7 and 3​

Answers

Answer:

44

Step-by-step explanation:

The difference between 15 and 9 is 6. 5 times the sum of 7 and 3 is 50 because 7+3=10 and 10 times 5 is 50. So if you subtract the difference between 15 and 9 from 50 you get 44.

Correct and fastest answer gets brainest! 12 divided 2/5

Answers

30

Answer:

12÷2/5=12*5/2=30 is a required answer

Answer:

30

Step-by-step explanation:

the tens digit of a two digit number is 5 greater the units digit. If you subtract double the reversed number from it, the result is a fourth of the original number. Find the original number.

Answers

Given:

The tens digit of a two digit number is 5 greater the units digit.

If you subtract double the reversed number from it, the result is a fourth of the original number.

To find:

The original number.

Solution:

Let n be the two digit number and x be the unit digit. Then tens digit is (x+5) and the original number is:

[tex]n=(x+5)\times 10+x\times 1[/tex]

[tex]n=10x+50+x[/tex]

[tex]n=11x+50[/tex]

Reversed number is:

[tex]x\times 10+(x+5)\times 1=10x+x+5[/tex]

[tex]x\times 10+(x+5)\times 1=11x+5[/tex]

If you subtract double the reversed number from it, the result is a fourth of the original number.

[tex]11x+50-2(11x+5)=\dfrac{1}{4}(11x+50)[/tex]

[tex]11x+50-22x-10=\dfrac{1}{4}(11x+50)[/tex]

[tex]40-11x=\dfrac{1}{4}(11x+50)[/tex]

Multiply both sides by 4.

[tex]160-44x=11x+50[/tex]

[tex]160-50=11x+44x[/tex]

[tex]110=55x[/tex]

Divide both sides by 55.

[tex]\dfrac{110}{55}=x[/tex]

[tex]2=x[/tex]

The unit digit is 2. So, the tens digit is [tex]2+5=7[/tex].

Therefore, the original number is 72.


SOMEONEEEE HELPPP MEEEE OUTTTTTTT!!!!!

Answers

Answer:

4/3

Step-by-step explanation:

Since this is a right triangle,

tan C = opp side / adjacent side

tan C = 36/ 27

tan C = 4/3

Given: x - 7 > -2.

Choose the solution set.

A. {x | x R, x > 14}
B. {x | x R, x > -5}
C. {x | x R, x > 5}
D. {x | x R, x > -9}

Answers

SoLuTiOn~

Given set:- x - 7 > - 2

Solving It:-

x - 7 > - 2

x > -2 + 7 [Here '7' is greater than '-2' So Sign Changes To Positive]

x > 5

So Correct Solution Set Will Be

Option C= {x | x R, x > 5}

Hope This Helps You

When AG = 16 ft, find the area of the region that is NOT shaded. Round to the nearest tenth.​

Answers

Answer:

730.88

Step-by-step explanation:

Area of the entire circle = pi * r^2

r = 16

Area = 3.14 * 16^2

Area = 803.84

1/4 of the circle contains the shaded area. It's area = 1/4 * 803.84

Area of 1/4 circle =

200.96

the area of the triangle

Area = 1/2 AG * G?

AG and G? are equal

Area = 1/2 * 16^2

Area = 128

Area of 1/4 circle - area of the triangle = area of the shaded portion

shaded portion = 200.95 - 128

Shaded Portion = 72.96

So the area of the unshaded part

unshaded = 803.84 - 72.96

Unshaded = 730.88

The function f(t) = 3 - t shows the cost of an ice cream sundae ($) with a different number of toppings (t). What is the slope of the function?

Answers

Answer:

-1

Step-by-step explanation:

are you sure this is the right equation ? it would mean that the more toppings the cheaper the ice cream.

but as it is written, the slope is -1.

the slope of a line is always the factor of the variable in the equation.

it is the ratio of y/x indicating how many units y is changing for a given change of x.

in our example here, if x changes 1 unit to the right (+1), then y changes 1 unit down (-1).

so, -1/+1 = -1

The slope of the function f(t) = 3 - t is -1

What is function?

"It defines a relation between input and output values."

What is the slope of the function?

"It is the rate of change of the dependent variable of the function with respect to that of the independent variable."

For given question,

We have been given a function f(t) = 3 - t

We need to find the slope of the function.

[tex]m=\frac{d}{dt}f(t)\\\\ m=\frac{d}{dt}( 3 - t)\\\\m=-1[/tex]

Therefore, the slope of the function f(t) = 3 - t is -1

Learn more about the slope of the function here:

https://brainly.com/question/9554035

#SPJ2

need some help with this​

Answers

Answer: The answer to that problem is the last equation (y = 4x - 7).

Reasoning: Since the slope is 4, it goes with “x”. The y-intercept is -7 and it goes to the end of the equation.

Answer:

y=4x-7

Step-by-step explanation:

here,

the equation of straight line in slope intercept form is;

y=mx+c

( m= slope

c= y-intercept )

soo..

the question has asked for slope 4 i.e. m=4

and y- intercept -7 i.e. c= -7

now.

the required equation is

y= 4x-7

mark me brainliest and follow me ... please

2/5(1/3x-15/8)-1/3(1/2-2/3x)

Answers

Answer:

[tex]\frac{16}{45}x-\frac{11}{12}[/tex]

Step-by-step explanation:

We are given that an expression

[tex]\frac{2}{5}(1/3x-15/8)-\frac{1}{3}(1/2-2/3x)[/tex]

We have to find the equivalent expression.

[tex]\frac{2}{5}(\frac{1}{3}x-\frac{15}{8})-\frac{1}{3}(\frac{1}{2}-\frac{2}{3}x)[/tex]

[tex]\frac{2}{5}\times \frac{1}{3}x-\frac{2}{5}\times \frac{15}{8}-\frac{1}{3}\times \frac{1}{2}-\frac{1}{3}\times (-\frac{2}{3}x)[/tex]

Using the the property

[tex]a\cdot (c-b)=a\cdot c-a\cdot b[/tex]

[tex]\frac{2}{5}(1/3x-15/8)-\frac{1}{3}(1/2-2/3x)[/tex]

[tex]=\frac{2}{15}x-\frac{3}{4}-\frac{1}{6}+\frac{2}{9}x[/tex]

[tex]=\frac{6x+10x}{45}+\frac{-9-2}{12}[/tex]

[tex]=\frac{16}{45}x-\frac{11}{12}[/tex]

[tex]\frac{2}{5}(1/3x-15/8)-\frac{1}{3}(1/2-2/3x)=\frac{16}{45}x-\frac{11}{12}[/tex]

Do not round your answer. Type in the number with the decimal. 15% of 25 is

Answers

Answer:

to add on to what the other dude said its 4

Step-by-step explanation:

Is the data set "color of the coal taken from a rail car" quantitive or qualitative? If it is quantitative is it descents or continuous?
A quantitative , discrete
B quantitative continuous
C qualitative
D neither

Answers

Answer:

It's A.)

Step-by-step explanation:

The correct answer is A

Given that PQ/ST = QR/TU= RS/US, select the postulate or theorem that you can use to conclude that the triangles are similar.

Answers

Answer: SSS Similarity Theorem (Choice A)

This is because we have three pairs of corresponding sides that form the same ratio, as shown by the given equation PQ/ST = QR/TU = RS/US.

That equation is basically the shorthand version of PQ/ST = QR/TU and QR/TU = RS/US combined together as one.

Logan was going to go to
Disneyland in America so he got
$450 from the bank. Then he
changed my travel plans, to go to
Eurodisney in Paris. How many
Euros will he have to spend?
Exchange Rate
GBP to Dollars £1: $1.50
GBP to Euros £1: €1.38

Answers

The answer is €414. Let me know if you need detailed solving

At a coffee shop, the first 100 customers'
orders were as follows.
Small
Large
Medium
Hot
un
48
22
Cold
8
12
5
5
What is the probability that a customer ordered
a large given that he or she ordered a cold
drink?
Rounded to the nearest percent, [? ]%

Answers

Answer:

people who ordered a cold drink=

[tex]8+12+5=25[/tex]

people who ordered a large cold drink= 5

[tex]probability= \frac{5}{25}[/tex] [tex]=\frac{1}{5}[/tex]

[tex]=\frac{1}{5} \times100=20~\%[/tex]

[tex]Answer: 20\%[/tex]

-----------------------

Hope it helps...

Have a great day!!

The probability that the customer ordered a large given that he or she ordered a cold drink is 5%.

What is Probability?

Probability is simply the possibility of getting an event. Or in other words, we are predicting the chance of getting an event.

The value of probability will be always in the range from 0 to 1.

Given the order of the first 100 customers in a coffee shop.

Number of customers who ordered a large which is also a cold drink = 5

Total number of customers = 100

Probability = Number of desired outcomes / Total number of outcomes

Substituting,

Probability = 5 / 100 = 0.05 = 5%

Hence the required percent is 5%.

Learn more about Probability here :

https://brainly.com/question/30034780

#SPJ7

What is the order of rotational symmetry for the figure?

Answers

Answer:

it should be 3

Step-by-step explanation:

I hope this help

A school in Delhi has its own flag, which is rectangular and divided into 4 rows and 6 columns. The word ‘SCHOOL' is inscribed in the first row, ‘FLAG' in the fourth row, and a purple coloured rhombus is in the middle

Answers

The question is incomplete. The complete question is :

A school in Delhi has its own flag, which is rectangular and divided into 4 rows and 6 columns. The word ‘SCHOOL’ is inscribed in the first row, ‘FLAG’ in the fourth row, and a  purple colored rhombus is in the middle, as shown in the given figure.

Based on this picture of the school flag, answer the following questions:  

(a) What fraction of the flag has the purple rhombus?

Solution :

From the figure, the total number of squares in the flag of 4 rows and 6 columns are : 4 x 6 = 24 blocks

The purple colored rhombus is made on the half of the 4 blocks.

i.e. [tex]$\frac{1}{2} \times 4 = 2\ \text{blocks}$[/tex]

Therefore, the fraction of the flag that has the purple rhombus is :

[tex]$=\frac{2}{24}$[/tex]

[tex]$=\frac{1}{12}$[/tex]

Other Questions
Devy likes to learn! Could someone please tell me how to answer this question?If f(x) and g(x) are inverse functions of each other, which of the following shows the graph of f(g(x))?On a coordinate plane, a straight line has a positive slope and goes through (negative 2, negative 1), (0, 0), and (4, 2).On a coordinate plane, a straight line has a positive slope and goes through (negative 3, negative 3), (0, 0), and (3, 3).On a coordinate plane, a straight line has a negative slope and goes through (negative 4, 2), (0, 0), and (4, negative 2).On a coordinate plane, a straight line has a negative slope and goes through (negative 3, 3), (0, 0), (3, negative 3). which of the following is equal to square root ^3 square root 2 ANX == [?]30XBCangles are not drawn to scale in 3-5 sentences thoroughly explain how the author choice of words regarding structure create tension or add meaning in the story "Condensed Milk" Find the midpoint of AC. What is the simplified form of V 100x35 ? Geometry, please answer question ASAP To study the mean respiratory rate of all people in his state, Frank samples the population by dividing the residents by towns and randomly selecting 12 of the towns. He then collects data from all the residents in the selected towns. Which type of sampling is used Cu 1: t trng l mi trng?A. Cung cp cht dinh dng, xyB. Gip cy ng vngC. Cung cp cht dinh dng, xy, ncD. Cu B v C PLS HELP ME ON THIS QUESGTION I WILL MARK YOU AS BRAINLIEST IF YOU KNOW THE ANSWER PLS GIVE ME A STEP BY STEP EXPLANATION!!The difference between the largest and smallest values in a data set is the ______________.A. meanB. modeC. medianD. range If the volume of the expanding cube is increasing at the rate 24 cm3 / min , how fast is its surface area increasing when the surface area is 216 cm2 ? Plz help.....Ill give you brainlyFind the sum or difference. 1 1 34 1-5 a. -122 + 42 b. -0.35 - (-0.25) I are these orders pairs a function, 0,92,8.4,76,68,510,4 An oscilloscope display grid or scale is called? Trade in the Vedic Age occurred through the bartering of goods for each other the exchange of coins for goods the use of written contracts as promises of payment the exchange of land and animals for money Choose the correct simplification of the expression 5x2(4x 6x2 3) Which option is the best point of comparison between the genres of horrorand comedy?A. The role of fear in the storyB. The importance of the pastC. Hijinks and absurd situationsD. The direction of a typical plot which questions can you answer based on information from this side of the page flying with the Arctic terns A stained glass window in a cathedral. Various pieces of colored glass form a religious scene.The above piece of art is a replica of what type of religious art?a.monogramc.stained-glass windowb.painting d.engraving Which lifestyle choice may contribute to the development of noncommunicable diseases?taking the stairseating fast foodexercising regularlyeating fruits and vegetables